LSAT For Dummies, 2nd Edition (2014)

Part VI. The Real Deal: Full-Length Practice LSATs

Chapter 18. Practice Exam 2: Answers and Explanations

This chapter has answers and explanations to the multiple-choice sections in Chapter 17, as well as an example essay for the writing sample (even though it isn’t scored) and an abbreviated key at the end of the chapter if you need to score your test quickly.

Section I: Logical Reasoning

1. B. This catalog is economical only when its normal prices are marked down more than occasionally.

The review must be assuming that it's impossible that the prices in this catalog can save customers money without being specially marked down. Choice (A) is wrong because the argument doesn't depend on the behavior of the catalog's competitors. Choice (B) looks correct. The review must assume this catalog's prices are cheap only when marked down. Choice (C) isn't right. When the catalog does mark down its prices, customers evidently save money. Choice (D) is wrong because this argument isn't about the catalog's competitors. Choice (E) doesn't work because the argument doesn't address mail-order purchasing as a whole, nor does it propose what the customers should do. Choice (B) is correct.

2. E. None of the pieces of evidence point directly to the conclusion, and all of them could lead to different conclusions.

The conclusion is that the settlement was destroyed in war. The evidence, though, doesn't seem that clear. It's not a circular argument, because the author doesn't state upfront that he thinks the settlement perished in war, so Choice (A) is wrong. Choice (B) doesn't work. Yes, the evidence is varied and the conclusion is unified, but that's what conclusions are supposed to do — bring together varied evidence into a single conclusion. Choice (C) is wrong because the argument doesn't contain any intermediate conclusions. Choice (D) is wrong because the argument doesn't make any historical conclusions. Choice (E) is the best answer; it's by no means certain that these bits of evidence point to one single conclusion.

3. C. The degree of erosion to which a coastline is subject is related to the shape of the sea bottom.

Choice (A) doesn't look right. If fetch is the length of the surface of the water, it shouldn't be related to the shape of the sea bottom. Choice (B) definitely seems wrong because the statements only state the factors (wind velocity and fetch) that influence wave size; there's nothing to suggest that wave size stays close to an average. Choice (C) does make sense because the impact of waves is related to the shape of the sea bottom, and the coast's erosion is related to the impact of waves. Choice (D) is wrong because the size of waves comes from wind and fetch, not the shape of the bottom. Choice (E) looks wrong, too. Wind velocity creates size of waves, size of waves affects impact, and impact affects erosion, so average velocity of wind playing no role in erosion doesn't make sense. Choice (C) is the best answer.

4. D. It is used to explain a consideration that may be taken to undermine the argument's conclusion.

The historian is arguing that for historians, seeing one event as an inevitable consequence of another is a mistake. The phrase in question explains that the historians’ position of looking back on events results in their erroneous belief that, because one event precedes another, a prior event necessarily causes the later event. Choice (A) is wrong; a “theoretical imperative” sounds like a requirement, and this statement is an explanation. Choice (B) isn't an illustration of the premise, which would be an example of a historian making this kind of mistaken attribution of causation. The statement doesn't support any claim, so Choice (C) is wrong. Choice (D) seems to make sense. The historian is explaining that, although historians sometimes see events as inevitable consequences of other events, that's just an occupational hazard of the historical profession and not justified. Choice (E) is wrong. The conclusion is that historians can't in fact assert that events follow one another inevitably. Choice (D) is the best answer.

5. D. When agricultural production is market-driven, it is likely to respond to rising demand by increasing production.

The speaker concludes that Chinese production will rise if demand requires it based on the premise that Chinese production is now market-driven. Clearly the author connects market-driven methods with matching supply and demand. Choice (A) isn't right because it isn't actually about the difference between global and regional markets but about China's transformation to a market economy. Choice (B) doesn't justify the conclusion but explains how things worked under China's older system. Choice (C) is wrong because the speaker isn't talking about local production. Choice (D) looks like the right answer. The speaker bases his conclusion on the principle that a market economy will respond to increased demand with increased production. Choice (E) isn't right. The speaker doesn't mention inefficiency or even compare a market-driven system to a centrally regulated one; he's only interested in the effects of China's transition between the two. Choice (D) is the best answer.

6. A. It is a specific example of a general condition described in the course of the argument.

The economist argues that, because patients need medical care and hospitals, regardless of what those services cost, hospitals and doctors rather than insurers bear the brunt of cost-containment measures; the MRI statement provides an example. Choice (A) is a good answer; the statement is a specific example of capital demands (MRIs and buildings) of the general condition of fiscal discipline described in the argument. Choice (B) doesn't work because the MRI statement doesn't counter an attack. Choice (C) isn't as good an answer as Choice (A). The author's claim or conclusion is that health insurers are still profiting from healthcare while doctors, hospitals, and patients are being increasingly squeezed, but the MRI statement doesn't indirectly support that claim. Choice (D) doesn't work. Patients’ needing treatment isn't a social side effect but a normal event that remains consistent, regardless of changing circumstances. Choice (E) is wrong; the MRI statement doesn't introduce the conclusion about the immunity of health insurers. Choice (A) is correct.

7. B. that no other explanations exist for the decreasing revenues from CD sales

To make her conclusion, the author must assume that PTP file-sharing was entirely or largely responsible for the drop in sales of CDs. She doesn't mention copyright issues, so Choice (A) is wrong. Choice (B) looks like the right answer. Choice (C) isn't at issue; the argument doesn't address the rights of musicians or producers to profit. Choice (D) is a possibility, but it's not as good as Choice (B). It's not obvious that the author specifically assumes that people who download music would purchase it on CD if they couldn't download it. People may have downloaded far more pieces of music than they would have purchased on CDs. Choice (E) doesn't really fit with the argument, which isn't about complex relationships in the music market but about how file-sharing has killed the record industry. Choice (B) is best.

8. B. fails to take into account any practical factors that may limit the number of redundant systems or practical trade-offs involved in increasing levels of redundancy

The engineer believes that the more redundant control systems an airplane has, the safer it will be. But what if having so many control systems can cause problems of its own or costs more than it's worth, either in economic or practical terms? Choice (A) is wrong; the engineer does think that simultaneous failure can occur, which is why he believes multiple systems are necessary. Choice (B) looks like it could be the answer because it addresses problems involved with redundancy itself. Choice (C) is wrong. The engineer is discussing control surfaces, so he's not obligated to consider other safety considerations. Choice (D) is wrong. The engineer discusses airplane safety and isn't obligated to apply these concepts to other projects. Choice (E) is also wrong because the argument doesn't depend on being able to evaluate relative levels of safety. Choice (B) is correct.

9. E. Allowing employees to take leave for family matters reduces absenteeism, improves morale, and surprisingly increases productivity because the employees who are granted leave tend to work much harder and more efficiently when they come back to work.

To weaken the argument, look for an answer showing that allowing family leave doesn't hurt productivity or perhaps even helps it. Choice (A) doesn't affect the argument because standard of living isn't an issue, and it doesn't mention workplace productivity. Choice (B) could arguably weaken the argument because it provides evidence that workers may not abuse the privilege of leave — fathers aren't taking family leave at all, which weakens the conclusion that workers would work less if they had leave. On the other hand, if taking paternity leave angers co-workers, that strengthens the conclusion that family leave hurts workplace morale, so this isn't the best answer. Choice (C) strengthens the argument by showing that FMLA leave costs the employer money. Choice (D) also strengthens the argument by illustrating the destruction caused by one employee leaving for a while. Choice (E) weakens the argument. If employers are worried about productivity and morale, this choice says that allowing leave actually increases productivity and morale. Choice (E) is the right answer.

10. A. Businesses profit from casino gambling because they take in money from local and visiting gamblers, whereas local individuals have more opportunities to lose money gambling.

Look for an answer that may explain why counties with casinos have thriving businesses but bankrupt individuals. Choice (A) looks right. Businesses take in money from gamblers (who aren't all local), and locals give their money to casinos. Choice (B) doesn't help. Casino odds may explain why gamblers would go bankrupt but not why businesses would profit. Choice (C) explains why businesses thrive but not why individuals suffer. Choice (D) explains gamblers’ losses but not businesses’ gains. Choice (E) may help explain why individual bankruptcy rates are so high in areas with casinos but does nothing to explain the profitability of businesses in those areas. Choice (A) is the best explanation.

11. E. Some food stamp recipients say they prefer the paper coupons because they can't tell how much money they have in their debit card accounts.

Eliminate answers that support the position that debit cards are better than paper coupons. Choice (A) supports the argument by showing how the new debit cards increase participation. Choice (B) supports it by showing how paper coupons were often used for fraud. Choice (C) supports the worker's position because the debit cards can be used only for approved items. Choice (D) supports it because it shows how the cards are much more convenient, which will increase participation. Choice (E) is the only answer that doesn't support the social worker's position; it presents a disadvantage of the debit cards. Choice (E) is correct.

12. C. It highlights the importance of this discovery because it disproves a long-held theory about hibernation.

This discovery of an animal that hibernates in hot weather may be groundbreaking, especially if previous scientific wisdom held that hibernation only happens in cold weather. Choice (A) is wrong because the belief being challenged isn't that primates never hibernate but that animals never hibernate in the heat. Choice (B) isn't right because the assertion isn't an accusation of any kind. Choice (C) makes the most sense because it's an important discovery. Choice (D) is wrong. The primatologist never disputes the conclusion that the behavior is in fact hibernation. Choice (E) doesn't work because the primatologist isn't setting up a rival theory in a deliberate ploy to attack it. Choice (C) is right.

13. C. To increase sales, having a famous actor in the title role of a film is more important than having an actor who can speak the film's language.

Wow, that's a gamble, casting an English-speaking actor in an Italian movie and letting him speak his part in English while everyone else speaks Italian. But the director must have his reasons; he clearly thinks that Lancaster's fame is a more valuable commodity for sales than an ability to speak Italian would be. He must not think that Italian audiences would refuse to see the film with Lancaster or he would never cast him, so Choice (A) is wrong. Choice (B) may be true, but actors’ objections don't seem to be the director's concern. Choice (C) is right — the director obviously believes fame is more important to box-office success than Italian ability. Choice (D) is beside the point. The director doesn't think coaching Lancaster in Italian is necessary. Choice (E) is also beside the point; maybe Lancaster would feel uncomfortable, but the director isn't concerned about that here. Choice (C) is correct.

14. E. The only substances that could eradicate all mosquitoes would also kill off many birds and beneficial insects.

Alberto suggests that, because losing a single species won't hurt an ecosystem, eradicating mosquitoes will cause no environmental problems. Look for an answer that disproves this. Choice (A) is a reason in favor of eradicating mosquitoes. Choice (B) doesn't strengthen or weaken the argument because it concentrates on the effect of restoring wetlands on the mosquito population rather than the effect that eradicating the mosquito population would have on wetlands. Choice (C) provides another strike against mosquitoes and may strengthen the argument. Choice (D) may suggest that mosquitoes do play a role in the ecosystem, but by itself it doesn't really hurt Alberto's argument. The answer could weaken the argument if it were more specific, such as providing evidence that mosquitoes are the only thing some birds eat. Choice (E) does weaken Alberto's conclusion. If the only way to eradicate mosquitoes would inevitably devastate birds and other insects, then Alberto can't suggest that his proposal would have no environmental costs. Choice (E) is right.

15. E. large numbers of people coming to a park prevent individuals from enjoying use of the park

The author assumes that large groups in parks are bad, but she gives no reasons for this claim; she just assumes that they're bad and shouldn't be in the park. She implies but doesn't directly state that large groups somehow interfere with all citizens’ equal rights to the park. Choice (A) is wrong because the author doesn't suggest that large groups are noisy or violent. She mentions no complaints, so Choice (B) is wrong. Choice (C) goes against her argument, so it's wrong. Choice (D) likewise weakens her argument, so it's wrong. Choice (E) is the only answer that makes sense. If she's assuming that large groups interfere with individual citizens’ enjoyment of the park, then the groups would be a problem. Choice (E) is correct.

16. D. Funds saved in FSAs must be spent during the plan year or forfeited.

Choice (A) looks like a reason why employees should use FSAs. Choice (B) offers an explanation of why there may not be widespread employer participation in FSAs, but it doesn't explain why many of those employees who have the opportunity to participate in FSAs choose not to. Cross out Choice (C) because it's another clear benefit of FSAs. Choice (D) is a drawback of FSAs and may well explain why few employees use them. Choice (E) describes a trend toward FSA participation and therefore doesn't explain why so few employees currently use FSAs. Choice (D) is the best answer.

17. C. A majority of residents of these six Latin American countries do not believe that democracy is necessarily the best form of government for them.

It appears from this argument that many Latin American people don't like their democratic governments. Choice (A) provides additional support for the argument but doesn't draw a conclusion from the statements provided. Choice (B) offers an explanation for the residents’ dissatisfaction with their governments but doesn't sum up the argument. Choice (C) makes sense as a conclusion; if the people are willing to substitute democratically elected leaders with a dictator, they must not be impressed with their leadership. Choice (D) introduces a side issue and is too specific to be a conclusion. Choice (E) supplies evidence that would support the conclusion but isn't itself a conclusion. Choice (C) is the best answer.

18. B. The relationship between salmonella and eggs has in fact changed over the years, justifying the recent caution with which people regard raw eggs.

The argument is about the changed risk of salmonella. In the past salmonella was only on the outside of eggs, but now it's inside the eggs themselves. Choice (A) can't conclude the argument because the author doesn't specifically state that salmonella has increased, only that its means of transmission has changed. Choice (B) makes sense as the conclusion; it sums up the argument's premise that changes in the way salmonella presents itself in eggs has altered the way people must handle eggs as food. Choice (C) isn't right. The author isn't suggesting that salmonella can't be serious, only that it's rarely fatal. Choice (D) doesn't quite work, because the author doesn't describe any change in egg production, only a change in the chickens producing them. Additionally, it doesn't address the egg preparation changes people have had to make. Choice (E) isn't right either. People in the past ate raw eggs safely because the eggs were uncontaminated, not because people were blissfully ignorant that the eggs contained salmonella. Choice (B) is best.

19. A. Operating systems with generous amounts of memory are less susceptible to crashing, even when applications are poorly written.

Okay, you want to find the four answers indicating that operating systems are responsible for the smooth functioning of applications and are able to somehow manage their memory problems. The best way to do this is by process of elimination. If you can find four answers that show the operating system handling applications’ memory issues, then the answer that's left over should be correct. Choice (B) helps the conclusion because it shows that operating systems are responsible for handling the memory used by individual applications. Choice (C) helps because it shows that operating systems can spot overuse of memory and stop it. Choice (D) helps because it tells you that programmers should know how to program an operating system that can prevent memory errors, which means all operating systems should be able to do this. Choice (E) helps the conclusion because it describes what an efficient operating system should be able to do. Choice (A) is the only answer that doesn't put responsibility for memory management on the operating system; adding memory to the computer evidently can let the operating system off the hook. Choice (A) is the right answer.

20. D. Ted's parents have stated that he cannot drive the station wagon unless it is to Alice's house. When his mother saw the station wagon parked at the mall, some miles away, Ted argued that Alice was not at home. Ted's mom pointed out that he ought, upon discovering that fact, to have driven straight home.

This question is a tough one, requiring lots of reading and thinking. Company A has a limited license to use a product but violates the license, claiming it's invalid. But there's another rule beyond the license that makes Company A's actions wrong, even if the license is invalid. So General Rule (GR) states an action can't be taken; Limited Rule (LR) provides an exception to GR in certain circumstances. Party A violates LR by taking action and says it's okay because LR is invalid. Party A is still wrong because action violates GR. Whew! Look for an answer that follows this pattern. Choice (A)'s scenario provides GR (warranty). Party A (consumers) took an action that appeared to be endorsed by another document. Party A successfully proved it didn't violate GR. Because Party A prevails, the pattern can't be the same. Choice (B) provides GR (no 13-year-olds are allowed into NC-17 movies). The ticket seller made a mistake, which isn't the same as authorizing an LR exception, so the pattern breaks down in this scenario. Choice (C) portrays the attempt of Party A (the manager) to create a LR for the GR of the distilled liquor license. The authority states that the LR doesn't exist. This situation isn't the same as the original. Evaluate Choice (D). The GR is that Party A (Ted) can't drive the station wagon. The LR is that he may drive it to Alice's house. Party A violates LR but says it's okay because LR wasn't available. The authority (Mom) says Party A is still wrong because he violated GR. Even though Ted's situation is very different in subject matter from the original, it follows the same rule pattern. Choice (D) is likely the best answer. Choice (E) doesn't establish an LR for Party A to violate. The celebrity didn't grant permission to the magazine to photograph him in any circumstances, so the pattern is different. Choice (D) is correct.

21. B. No single written edition of a Greek epic can claim to represent the “original” version.

The author's conclusion is that texts of poems differed from place to place because of the changes in oral transmission of the same stories. Choice (A) is clearly wrong because the scholar isn't suggesting that each classical epic had one original text. Choice (B) looks right because the scholar argues that there can't be any “original” text but instead many written versions of the same epics. Choice (C) doesn't make sense. The scholar doesn't mention narratological complexity. Choice (D) doesn't work, either, because the scholar doesn't imply that people stopped transmitting epics orally after the poems were written down but in fact suggests the opposite — that they continued to compose even after they began transcribing. Choice (E) is totally wrong; the scholar gives no suggestion of political influence in the argument. Choice (B) is correct.

22. E. If people see online images of items in the museum's collection, they will no longer be interested in seeing the collection with their own eyes.

The curator seems to assume that if people see the images online, they won't have any interest in visiting in person. Choice (A) isn't the point because the curator isn't worried about damaging the images. Choice (B) doesn't work because the curator doesn't mention a concern for decreased revenue. Choice (C) likely isn't the curator's concern. He isn't specifically worried about the extent of online distribution but rather its effect. Check the remaining answers to see whether you have a better option. Choice (D) isn't his concern, either, because he doesn't mention quality issues. Choice (E) is the best answer. The curator is worried that online publication of the images will remove the incentive to visit the actual museum in person. Choice (E) is best.

23. A. Spousal and marital difficulties were formerly responsible for a large number of premature returns from foreign assignments.

If helping spouses has improved expatriate retention by such a huge amount, then unhappy spouses must have previously been responsible for lots of premature returns. Choice (A) looks like a good answer. If unhappy spouses contributed to employees’ leaving international assignments, helping spouses adjust would improve the situation. Choice (B) is wrong. If spouses are already thrilled with the international experience, their dissatisfaction is unlikely to contribute to employees’ leaving their overseas posts. Choice (C) would support the argument, but it's too specific to be a necessary assumption on which the conclusion depends (there could well be other reasons why spouses are dissatisfied). Choice (D) doesn't explain why helping spouses has improved retention. Choice (E) provides an example of what companies are doing to help spouses but isn't the assumption that links the argument's premises to the conclusion. Choice (A) is the best answer.

24. E. You catch more flies with honey than you do with vinegar.

Now, this is a nice little question — a story and some proverbs. You have to decide which proverb — principle — matches the traveler's experience. A stitch in time saves nine means that early correction of problems prevents them from getting bigger; that doesn't really fit here, so Choice (A) is wrong. Don't price an unborn calf means you shouldn't depend on an event that hasn't yet occurred; that's not right, so Choice (B) is wrong. Choice (C) may work because the traveler does carry his computer with him, but it doesn't explain the comparative reaction of the clerk. Choice (D) is wrong because the traveler isn't borrowing or lending anything. Choice (E) is the best answer; because he was nice (honey) to the clerk (the fly), she was nice to him and punished the rude (vinegar) travelers by dawdling.

Section II: Reading Comprehension

1. D. Manning's work exemplifies how biography can be a powerful tool for a historian of science, who can use the genre to explore the effects of politics, economics, and emotions on the direction of scientific development.

This passage is mainly about the effectiveness of biography as a genre for exploring the history of science and the importance of analyzing scientific discoveries from a historical perspective. The business about Black Apollois just an example the author uses to illustrate his point. So Choice (A) is wrong because the passage's main point isn't the importance of Ernest Everett Just. Choice (B) is a point the author makes in the last paragraph, but it's not the entire passage's main point. Choice (C) appears in the second paragraph, but once again, it doesn't cover the whole passage. Choice (D) looks like the right answer; it sums up the overarching theme of the passage. Choice (E) is wrong because, like Choice (A), it focuses too much on Black Apollo. Choice (D) is correct.

2. A. illuminate the effects of social forces on scientists in a way that scientists themselves are unlikely to do

According to the author, “One of the central principles of the history of science, indeed a central reason for the discipline, is to show that science is a product of social forces.” That makes Choice (A) look like a very good answer. Choice (B) isn't exactly right. The author says people can learn scientific theories by reading the work of the scientists themselves. The drawback is that the picture given by scientists is incomplete because it ignores historical context. Choice (C) is also imprecise. Historians of science do write biographies, but biographies are just one way to accomplish their main goal of revealing the social forces behind scientific discovery. Choice (D) is wrong because the author never suggests that historians of science want to influence scientific research. Choice (E) is likewise wrong. History of science exists to analyze science as a part of society, not to make science palatable to non-scientists. Choice (A) is right.

3. C. to explain why biography is both a popular historical genre and a powerful medium for explaining the significance of scientific discoveries

The second paragraph contains a discussion of biography as a historical genre and lists its many advantages. Choice (A) isn't at all the main point. It just barely appears in the paragraph. Choice (B) is wrong; the author does believe biography is a good historical form for the historian of science. Choice (C) looks like a good answer. Choice (D) is incorrect; the author says that biographies are very good for teaching children. Choice (E) never appears anywhere. Choice (C) is right.

4. A. One of the best ways to come to an understanding of the realities of race relations and scientific development in the 20th century is to read an in-depth account of the life of one of the people who lived and worked in that world.

Choice (A) looks pretty good; this is in fact what the author has been saying about the history of science. Choice (B) is wrong. The author doesn't think historians should glorify their subjects and notes that Manning doesn't glorify Just. Choice (C) is also wrong. The author doesn't imply that a scientific history should downplay science simply because it's “history.” Choice (D) isn't right. The author explicitly says that Just wasn't the most significant scientist of his time. Choice (E) contradicts what the author says in the second paragraph. Choice (A) is the best answer.

5. D. Just's daily experiences illuminate the conditions characterized by both scientific research and racial relations during his lifetime.

Here's what the passage says: “A comprehensive appreciation of the conditions that Just faced in his daily work offers a powerful lens through which to examine the development of science and racial boundaries in America.” Choice (D) looks like the answer that matches best with this statement. The other answers are all true, but they're also incidental, facts that add up to a bigger picture but by themselves aren't enough to create a significant history.

6. E. the straightforward organization of a biography, which follows the course of the subject's life

A biography is a story of a person's life; the format can't change that much. The author says, “Biographies simply tell a story.” The phrase “simplicity of form” doesn't refer to language, so Choice (A) is wrong, nor does it mean page design, so Choice (B) is wrong. It doesn't mean simple writing style, so Choice (C) is wrong. Choice (D) is wrong because the author suggests that a biography's power exists despite its simplicity. All the author means is that biographies have a standard format, which is fairly consistent from book to book; that makes Choice (E) correct.

7. C. SLS and SLES are detergents that are commonly used in personal care products because they are effective and safe, despite unsubstantiated rumors to the contrary.

This passage introduces the reader to a couple of detergents commonly used in numerous household products. It describes how they work and mentions a few hazards associated with them. The reason the author mentions those hazards in the second paragraph is to get the facts in ahead of the risks that are solely based on rumor, because her point in the last paragraph is that many of the things people fear about SLS and SLES aren't based on fact. She obviously thinks SLS and SLES are safe as they're commonly used and believes that approval by the FDA and other scientific organizations is sufficient proof of this safety.

Choice (A) is a possible answer, but it ignores the discussion of Internet detractors, so it doesn't cover the entire passage. Look for something with a more global application. Choice (B) is wrong. The presence of “despite” suggests that the author thinks incorporating SLS or SLES into personal care products is irresponsible or dangerous, which isn't justified by the passage. Choice (C) looks like a better answer than Choice (A) because it incorporates more of the passage's information. Choice (D) is wrong because this passage isn't about the Internet rumors but about counteracting them. Choice (E) is wrong. It's true that the author doesn't have a problem with using SLS in both shampoo and engine degreasers, but that's not the main point of the passage. Choice (C) is the best answer.

8. B. shampoo, toothpaste, bathroom cleaners, and engine degreasers

You have to read carefully to answer this one. Don't assume any product contains the substances unless the passage tells you so. If you prefer, you can underline or circle substances that the author mentions in the passage in the answer choices — that makes it easier to see the ones that appear in the text. Choice (A) is wrong because the author never mentions mouthwash, sunscreen, or hair dye. Choice (B) looks good. They're all mentioned in the passage. Choice (C) is wrong because the passage doesn't mention engine lubricants. Choice (D) is wrong because moisturizer and baby wipes don't appear. Choice (E) is wrong because fabric softener isn't in the passage, though you could probably infer that SLS and SLES are found in kitchen cleaners and laundry detergents. Choice (B) is correct.

9. E. refute claims that SLS and SLES are dangerous

The author says that the rumors about SLS and SLES are absurd and unsubstantiated and “the FDA has approved the use of SLS and SLES in a number of personal care products.” That means she's using FDA approval as evidence of the substances’ safety. Choice (A) looks like a possible answer, though it doesn't mention the author's suggestion that the FDA approval implies safety, so it misses the reason why the author brings up the FDA. Choice (B) is wrong because even though the author thinks that the FDA has the best interests of consumers in mind, that isn't the reason why the author mentions the FDA. The purpose of mentioning the FDA is to provide evidence debunking the Internet myths. Choice (C) doesn't work because the author isn't in fact suggesting that FDA approval of putting SLS and SLES in personal care products means that manufacturers don't have to test these products for safety. Choice (D) is wrong; the author isn't criticizing the FDA in any way — quite the contrary. Choice (E) works the best because the author mentions the FDA to achieve the larger goal of debunking the Internet rumors. Choice (E) is correct.

10. D. burning eyes, burned skin after long exposure, and diarrhea if ingested in large quantities

Read the passage carefully. The answers to this question appear in the second paragraph, not in the third paragraph, which lists risks that haven't been proven. Skim the answer choices to cross off anything that appears in the third paragraph. Choice (D) is the right answer. Every other choice contains ailments that Internet rumors have associated with SLS and SLES but that haven't been substantiated.

11. E. a description of SLS and SLES and their uses; known risks of SLS and SLES; criticisms aimed at SLS and SLES by detractors on the Internet; evidence that SLS and SLES are safe and the rumors unfounded

Look for an answer that could function as an accurate ordering of all paragraphs in the passage. Choice (A) isn't quite right because it leaves off the subject of the first paragraph. Choice (B) is wrong. The passage doesn't contain any anecdotal accounts of SLS injuries. Choice (C) doesn't work because the first paragraph doesn't describe the chemical composition of SLS and SLES. Choice (D) isn't quite right because it doesn't mention the known and unsubstantiated risks associated with the substances. Choice (E) is the best answer because it follows the structure of the passage closely.

12. E. to explain why some people fear SLS and SLES and to list the diseases that Internet rumors have linked to the substances

The third paragraph discusses the Internet rumors that hold SLS and SLES responsible for a host of ailments without providing proof. The author obviously wants to discredit these rumors; that's what the last sentence is all about. She's not criticizing, so Choice (A) is out. She's not describing the substances — that's in the first paragraph — so Choice (B) is out. She doesn't believe these risks are real, so she's not warning anyone of anything, and Choice (C) is out. She makes no proposals of alternate substances, so Choice (D) is out. Choice (E) is the best answer here.

13. A. It is unreasonable for people to be afraid of substances that have been deemed safe by the FDA and several other major organizations, and that have a long history of safe use, simply on the basis of unsubstantiated rumors.

Choice (A) looks like a good possibility. The author does seem to think it's silly to believe rumors about substances that people have been using safely for years. Choice (B) doesn't quite work. The author does trust the FDA but makes no mention of its sources of funding or mission. Choice (C) is wrong. While the author clearly thinks that some Internet information isn't trustworthy, the passage doesn't provide sufficient information for you to infer how she feels about information available about other health topics. For example, she could easily think that the Internet supplies good information on heart disease. Choice (D) doesn't work. The author does think SLS and SLES are cheap and effective surfactants and emulsifiers, but she doesn't think that they're unsafe to use in products that contact human skin. Choice (E) is wrong. The author probably wouldn't want to prevent manufacturers from using SLS and SLES in their personal care products, but there's no reason to believe that she thinks enough protests could stop this inclusion. She doesn't mention protests anywhere in the passage. Choice (A) is the best answer.

14. D. the amount of time that has passed since the eyewitness experienced the event

Answer this question by eliminating answers that Passage A indicates have been researched to discover their effect on the accuracy of eyewitness testimony. The third paragraph comes right out and tells you that race, gender, and age have been studied, so Choices (A), (B), and (E) are out. The second paragraph states that “much research” has been done on the effects of witnesses who are asked misleading questions. A misleading question is a type of question, so you can infer that the effect of the question type on eyewitness accounts has been studied and cross out Choice (C). You can reasonably assume that the amount of time that passes between an event and an eyewitness's account of that event would affect the accuracy of the testimony, but the question doesn't ask for your reasonable assumption. Because Passage A doesn't mention any studies conducted to see how time affects eyewitness testimony, Choice (D) offers the exception. Remember to answer questions based solely on information in the passage, regardless of any outside or personal knowledge you may have on the subject matter.

15. B. Eyewitness testimony is often flawed because it is influenced by a variety of factors.

The best answer incorporates a point suggested by both passages. Eliminate answers that can be supported by only one of the passages. Passage A makes the statement that there's no difference between the accuracy of male and female testimony, but Passage B doesn't discuss the role of gender, so Choice (D) is supported by only one passage and can't be right. Although both passages mention that eyewitness testimony is affected by a number of variables, only Passage B discusses the importance of determining an error rate. Passage A doesn't mention the establishment of an error rate for testimony, so you can't assume Passage A supports Choice (E). Neither passage discusses how memory improves or declines over time, so Choice (C) is out of contention. That leaves Choices (A) and (B). Choice (A) doesn't seem likely. The first line of Passage A states that “there are many factors that may account for mistaken eyewitness identification” and then goes on to describe the research of these factors, which implies that mistaken eyewitness identification occurs frequently enough to warrant significant study. Passage B stresses the science of memory and human cognitive abilities and states that they're “not perfect.” In the third paragraph, the author of Passage B points out that establishing an error rate involves an awareness of the many factors that affect eyewitness accounts. Neither passage suggests that eyewitness testimony is “highly” accurate, but both imply that accounts may be flawed by the influence of several factors or variables. Because it's a better answer than Choice (A), Choice (B) is correct.

16. A. How the lighting in a particular event affects the reliability of eyewitness identification is a variable that warrants a good amount of study.

Your job is to eliminate reasonable implications of Passage A. The passage tells you that one's race can affect how well one recognizes someone's face, so it implies that one's race may adversely affect the reliability of an identification. Cross out Choice (B). The passage justifies the statement in Choice (C). It categorizes the factors as system variables and estimator variables. So Choice (C) is wrong. The passage states that the majority of research has gone into studying system variables, and the makeup of a lineup is a system variable. So you can reasonably infer Choice (D) from the statements in Passage A. The passage tells you that the way questions are worded is a system variable, and the judicial system has control over system variables. Therefore, the author of Passage A must think that the judicial system has control over whether a witness is asked misleading questions. Because Choice (E) is wrong, Choice (A) must be the answer. Passage A states that lighting can affect an eyewitness identification, but because lighting is an estimator variable over which the judicial system has little control, it's unlikely to receive much research. The passage states that system variables receive the majority of study. The best answer is Choice (A).

17. E. to understand how memory and human cognitive abilities are affected by a variety of different factors

Both passages discuss memory and how it's affected by different factors; what differs is the factors they discuss. Passage A covers variables that include the age, race, and gender of eyewitnesses and the wording of the questions they're asked. Passage B emphasizes the complexity of memory and cognitive abilities. Rule out choices that pertain to one passage but not the other. Choice (A) is a concern of Passage B but isn't mentioned in Passage A, so it's wrong. Choices (C) and (D) are important to Passage A but not to Passage B. Choice (B) isn't a goal of either passage. Though Passage B does indeed mention episodic memory, aside from defining the term, the passage doesn't show how the concept contributes to judicial proceedings. By process of elimination, the best answer is Choice (E). Both passages deal with the complex factors that affect cognitive ability and memory as they relate to the accuracy of eyewitness testimony.

18. D. Both passages concern improving eyewitness accuracy, but Passage A focuses on controlling variables and Passage B concentrates on understanding the science behind human recollection.

You can eliminate some answers quickly because they aren't true. Choice (A) is wrong because Passage B doesn't discuss witness questioning and therefore doesn't dismiss its importance. Choice (B) isn't right because how the judicial system controls certain variables is a concern of Passage A, not Passage B. Eliminate Choice (E) because both passages deal with how research can improve eyewitness accuracy: through research of system variables for Passage A and the establishment of an error rate in Passage B. Choice (C) may be tempting because Passage A mentions the effect of race on eyewitness identification, but the way the question is worded implies that Passage B does indeed discuss race, at least to some degree, which is inaccurate. Choice (D) is the only option that appropriately defines a noticeable difference between the content of the two passages. The primary focus of Passage A is the variables that affect the accuracy of eyewitness identification. Passage B is more concerned with the workings of the human mind and how this knowledge can be used to establish an error rate for witness testimony. Choice (D) is best.

19. E. Lighting issues and the length of time someone witnessed an event are examples of system variables.

This question asks you for the statement that isn't supported by either passage. Eliminate answers that appear in either of the two passages. The second paragraph of Passage B states that the Daubert case argued for the need of an established error rate, so Choice (D) is easy to eliminate. Choice (A) is a premise of both passages; they both state that human memory doesn't get it right every time. Passage B says that eyewitness memory should make “guilty people seem more likely to be guilty,” and Choice (C) seems to paraphrase that statement. Passage B's first paragraph states that cognitive abilities are incredible and supports that statement with the assertion that visual, auditory, olfactory, tactile, and taste information synchronizes with past information to bring that information into the present, so the statement in Choice (B) is supported by Passage B. If you thought Choice (E) was supported by Passage A, you confused system variables with estimator variables. Lighting issues and the length of time someone witnessed an event are actually examples of estimator variables. Choice (E) is the answer that neither passage supports.

20. C. the philosophical origins of public schools in 18th century Germany and the transformation in educational thinking in the 19th century

This passage is about the origins of public education and the changes that occurred in educational philosophy in the first century of public schools; the whole thing is set in Germany. Choice (A) doesn't cover the whole passage; the political message seems to apply only to the first half. Choice (B) also focuses on just the first half and so isn't the passage's primary point. The final paragraph doesn't focus on exploitation at all. Choice (C) conveys the passage's overarching theme. Choice (D) is wrong because the passage doesn't get into modern educational practices. Choice (E) doesn't cover the whole passage, just the first part of it. Choice (C) is correct.

21. C. They were indifferent to the well-being and needs of their workers, caring only to maximize production and profits no matter what it cost their employees.

The author tells you that textile mill owners exploited their workers badly enough to incite revolts and that they embraced the concept of schools in the hopes that it would make the workers more docile. Choice (A) is quite wrong. The first schools weren't created to help the students so much as to help the nobles. Choice (B) could well be true, but the passage doesn't discuss it. Remember, all correct answers must not stray too far from the text. Choice (C) fits well with what the passage says about the owners. It does appear that they were indifferent to the well-being of their workers. Choice (D) isn't quite right. The passage doesn't specifically tell you that they were all aristocrats. Although some of them may have believed their authority was divinely ordained, you can't assume that was true of them all, nor is there any reason for you to assume that they cared about nurturing their workers. Choice (E) is wrong. The passage doesn't contain anything about the factory owners being patriotic. Choice (C) is the best answer.

22. B. Eighteenth-century schools were concerned primarily with teaching working-class children to accept their fate and love their ruler; 19th-century schools began to focus on developing the full human potential of students.

The educational difference between the two centuries was philosophical. Schools in the 1700s were meant for workers and intended to instill patriotism and gratitude toward the government into their students, but schools in the 1800s aspired to develop children to their full potential. Choice (A) isn't right because 18th-century schools had nothing to do with efficient textile mills. Choice (B) looks like a very good answer. Choice (C) doesn't work because 18th-century schools were for the children of workers, not the aristocracy, and in the 19th century, no one had to spin anymore. Choice (D) is tricky because it's very close to being correct, but the passage doesn't tell you that 19th-century schools aspired to create free-thinking students in general (though academic freedom was prized for advanced students), so it's wrong. Choice (E) may actually be true — it sounds like some parents of Spinnschulen children didn't like the schools if truant officers were necessary — but that's not the main difference between the centuries, and the passage doesn't really address this point. Choice (B) is the right answer.

23. A. riots and other forms of violence against the owners of textile factories by peasants unhappy at their treatment

Look at the sentence after the one that mentions increasing levels of unrest. It says that the rulers wanted “to channel the energy of restless peasants into something that would be less dangerous to the throne than riots.” So “unrest” must mean riots and other violent uprisings by workers who disliked their lot in life. That would be Choice (A). None of the other answers work. Choice (E) is tempting because it concerns young people, and this passage is about schools, but the statement doesn't restrict the violence to young men. Choice (A) is correct.

24. E. They liked the idea because it would make the peasantry more complacent and accepting of their fate, which would help keep the aristocracy safe in their prosperity.

According to the passage: “Aristocrats liked this idea. They liked the thought of schools making peasants more docile and patriotic, and they appreciated the way state-run schools would teach children of lower social classes to accept their position in life.” Choice (A) is wrong because the schools were intended to do just the opposite; educating workers was supposed to make them more docile, not more violent. Choice (B) doesn't work because the passage never mentions aristocratic resentment of taxes. Choice (C) may be a true statement, but the passage doesn't directly come out and say it. You know nothing of the aristocratic opinions of Schlabrendorff himself. Choice (D) is wrong because the passage doesn't tell you that the education would be specifically religious. Teaching children that their “lot was ordained by god” doesn't mean that the entire curriculum was religious. Choice (E) is the most suitable answer to this question.

25. B. to take away the authority of parents and replace it with state power over children and citizens

The truant officers were meant to take away parental authority over children and replace it with state control. Choice (B) is the most accurate answer. The truant officers weren't there to make sure every child was educated, so Choice (A) is wrong. They didn't help or indoctrinate parents or children, nor did they recruit boys into the army, which nixes Choices (C) and (D). Though they may have indirectly assisted the king and his administration in the compliant-citizen project, which would make Choice (E) a possible answer, but it's much less specific than the answer given in Choice (B). Choice (B) is best.

26. D. People learn best in an environment that respects their individuality, affords them freedom, and incorporates a variety of aspects of learning, such as physical movement, manual skills, and independent exploration.

Nineteenth-century educational theorists believed in nurturing innate abilities and using holistic techniques. That's not Choice (A). In fact, Choice (A) is just the opposite of what experts thought in the 1800s. Choice (B) is wrong because nothing in the passage mentions religion. Choice (C) isn't right. The passage never suggests that the state has an interest in an educated citizenry, just an interest in a docile and patriotic one, and that wasn't the prevailing view in the 19th century anyway. Choice (D) looks like a perfect answer to this question. Choice (E) isn't right because the passage never mentions that mothers were good teachers for their own children. That makes Choice (D) correct.

Section III: Logical Reasoning

1. A. Conservative Protestants tend to continue attending services even when they donate large amounts.

The economist is using information about a trend to make a generalization that applies to all people. An answer that offers an exception to the rule would weaken the argument, such as evidence of some people who donate a lot and still attend plenty of services. Choice (A) looks like just such an answer. Choice (B) strengthens the argument. Choice (C) shows how the argument applies to many denominations, which doesn't weaken it. Choice (D) is largely irrelevant. Choice (E) is a more detailed restatement of the argument. You can eliminate the other four options, so Choice (A) must be correct.

2. E. Most babies are born to mothers under the age of 35.

So the odds go up for older women, but the numbers are bigger for younger women. That must mean that the total number of children being born to the two groups must be very different. If younger women have more babies with Down syndrome despite lower odds, many more of them must be having babies. Choice (A) doesn't reconcile the discrepancy and, if anything, would lead you to expect more women in their forties would have babies with Down syndrome. Choice (B) doesn't affect the question. Choice (C) doesn't contain any info that would help with the apparent paradox because it doesn't report which age group the information about mothers’ decisions applies to. Choice (D) doesn't work because testing only provides information about which fetuses are affected, not why younger women produce the most Down syndrome children. Choice (E) is the best answer. Knowing that the majority of babies are born to women under 35 would explain why they also have the majority of Down syndrome babies.

3. B. It implies that if a consumer purchases these shoes and they hurt her feet, the company will refund her purchase price.

Choice (A) is incorrect because you have no reason to assume from the argument that advanced podiatric design is in fact essential to comfort. Choice (B) looks like a good answer. The guarantee implies that customers can get a refund if they don't experience the claims made in the rest of the argument. Choice (C) is wrong. The argument never says that everyone loves the shoes. Be wary of answer choices that contain extreme language, such as “all customers.” Choice (D) is wrong because neither the argument nor the key sentence criticizes other shoes. Choice (E) definitely doesn't apply to that sentence, which doesn't mention podiatrists. Choice (B) is right.

4. A. Many Cubans avoid financial hardship because they rely on cash and goods brought to them by relatives who previously could visit once a year and bring in a maximum of $3,000.

This argument's conclusion has two elements. To weaken the argument, look for an answer suggesting that these regulations either won't hasten the fall of Cuba's dictatorship or won't benefit all Cuban people. Choice (A) is an example of how the regulations could hurt Cubans, so Choice (A) looks like a good answer. Choice (B) merely restates one of the premises and doesn't weaken their link to the conclusion. Choice (C) describes Cuba's government but doesn't specifically weaken or strengthen the argument because it fails to mention the effect of the new restrictions. Choice (D) isn't a good answer because it doesn't show how the administration is wrong about the ban's effects (it could pander and still achieve its goals). Choice (E) may strengthen the argument — if Cuba's regime is benefiting from visitors, limiting them would hasten the dictator's demise. Choice (A) is correct.

5. D. assumes without offering evidence that customers value scheduling flexibility enough to pay double the frequent-flier miles for it

The argument assumes that customers will like this change because they want more flexibility. If they don't in fact want flexibility, the argument becomes weaker. Choice (A) doesn't work because the airline isn't trying to suggest that customers with more miles deserve more seats. Choice (B) is wrong because there's no criticism here. Choice (C) may be a true statement, but it doesn't relate to the conclusion about the way customers view the change rather than how airlines are affected by frequent-flier awards. Choice (D) looks like a good answer. The airline is simply assuming that customers will appreciate this change, when in fact many of them may prefer to get less flexible seats for fewer miles. Choice (E) doesn't work; the airline isn't encouraging upgrades. Choice (D) is the best answer.

6. C. Across all income levels, women have on average higher educational attainments than men.

Choice (A) isn't an assumption but is in fact stated in the argument. Choice (B) isn't necessarily true. The sociologist isn't suggesting that women are choosing wealthy, educated men for their physical attractiveness. The concept in Choice (C) plays an important role in the argument; if even the poorest women are generally better educated than their male peers, the marrying-equal or marrying-up practice will leave some of these men unmarried forever. Choice (D) may be true but isn't directly relevant to an argument about marriage. Choice (E) isn't something you can assume from what's stated in the argument and is irrelevant anyway. Choice (C) is the best answer.

7. E. Boss Stephanie constantly criticizes subordinate Kim's work, correcting it one way one day and then correcting it back to its original state the next. Kim meekly accepts the criticisms but spends every Thursday evening at a bar with co-workers making fun of Stephanie.

You're seeking an answer that shows someone who submits to abuse and then criticizes the boss in private. You can save time by reading the second half of the answer choices. The ones that match the subordinates’ behavior are the only ones you need to read entirely. The only answers that describe employees who complain about their bosses in private are Choices (D) and (E). Choice (A) doesn't work because Skylar does take formal action against his boss. Choice (B) is no good because David quits. Choice (C) doesn't work because Casey takes action within her company. Choice (D) is close, but it's wrong because Bill doesn't appear to be abusing Paul arbitrarily. The problem here is an inconsistent or incompetent boss more than a bullying boss. Choice (E) is the best answer.

8. D. Employers assume that high school graduates generally have a much higher level of mastery of academic subjects than those who earn GEDs.

The argument suggests that a GED is just as good as a high school education; look for an answer that contradicts that. Choice (A) doesn't work. You don't want evidence showing the benefits of earning GEDs. Choice (B) doesn't pose a problem. If universities accept GEDs, that's more evidence that they're as good as diplomas. Choice (C) actually strengthens the argument. Choice (D) does weaken it. If a GED might put one at a disadvantage on the job market, that's a reason to stay in school. Choice (E) doesn't strengthen or weaken the argument. Choice (D) is right.

9. D. A ban on over-the-counter sales of cough medicines containing codeine reduced incidences of codeine overdoses by 50 percent in two years’ time.

Look for an answer that supports the claim that a ban on over-the-counter DXM sales will protect young people. Choice (A) describes the bad effects of DXM, which bolsters the premises of this argument but doesn't prove that a ban on OTC sales will help curb overdoses. Choice (B) is irrelevant because the argument isn't about the addictive properties of DXM but rather its harmful effects when taken in large dosages. Choice (C) is just more information on how teenagers abuse DXM, not how a ban on selling it would help stop the abuse. Choice (D) supports the claim that banning OTC sales would help. It worked with codeine, so it should work with DXM. Choice (E) is evidence of why DXM is dangerous but not why banning OTC sales would work to prevent abuse. Choice (D) is correct.

10. A. A city on the coast is more vulnerable to being struck by a hurricane than an inland city.

Choice (A) looks like a very good answer. If hurricanes need to be over water to maintain their strength, inland cities must be safer from them than coastal cities. Choice (B) doesn't necessarily have to be true. Hurricanes form over tropical water, but the argument doesn't suggest that they can't go into colder water. Choice (C) happens to be true, but the statements don't mention anything about the Atlantic specifically, so you can't infer this. Choice (D) doesn't follow because the argument says hurricanes lose strength over land, but it never mentions how many miles inland a hurricane would have to move before it loses most of its power to do damage. Choice (E) may be tempting, but you can't assume it. The argument gives you no basis on which to make distinctions between the two seasons. Choice (A) is right.

11. E. assumes that no other parents will take the same stance against vaccination

This parent assumes his child will be safe from diseases because all other children are vaccinated against them. But what if they aren't? The parent doesn't criticize other parents, so Choice (A) is wrong. He isn't appealing to emotion or trying to persuade other parents, so Choice (B) can't be right. He doesn't mention the government, so Choice (C) is wrong. He doesn't mention the knowledge of other parents, so Choice (D) is wrong. But he does assume that everyone else will get his children vaccinated and is counting on that to protect his child. Choice (E) is the best answer.

12. C. equates freedom of expression by national leaders with proof that requiring students to mention God in the pledge is not an unjust imposition of religious beliefs

The speaker suggests that requiring teachers to lead students in the pledge with the words “under God” isn't an unconstitutional imposition of religious beliefs because national leaders mention God all the time. This comparison makes a mistake; the behavior of national leaders doesn't set standards for what is and isn't constitutional. Choice (A) is wrong — what other politicians do isn't the issue. Choice (B) doesn't work; the issue isn't public consensus but constitutionality. Choice (C) looks like a good answer; the real problem with this argument is that it confuses the acts of national leaders with proof that something isn't unconstitutional. Choice (D) is wrong because it's not true; the argument doesn't praise teachers. Choice (E) is wrong because it doesn't address the argument's logic. The argument isn't about what people believe but about the propriety of requiring everyone to make a statement of belief. Choice (C) is the best answer.

13. B. Europeans work shorter hours than Americans because higher income taxes remove any incentive they may feel to increase their take-home pay by working longer hours.

The speakers disagree about whether high income taxes are behind Europeans’ shorter work schedules. Choice (A) is wrong because they're not debating Europeans’ love of leisure. Choice (B) is the source of disagreement. Choice (C) is wrong. Neither one has suggested that the United States should raise income taxes. Choice (D) doesn't work. They're not comparing European and U.S. standards of living. Choice (E) may be tempting, but it's not the source of disagreement; it works better as supporting evidence for Preston's argument. Choice (B) is the best answer.

14. D. Getting citizens to use less energy than they currently do is desirable.

Choice (A) isn't the assumption. The argument suggests that the government should interfere with energy policies. Choice (B) doesn't work; the speaker advocates for the use of incentives, so he must think they're necessary. Choice (C) is wrong because the author never mentions 1970s policies, only that people in the 1970s managed to reduce their energy consumption in the face of a crisis. Choice (D) looks like an assumption the author makes; he would only suggest the use of incentives to reduce energy consumption if he thought reduced energy consumption was desirable. Choice (E) is wrong. The author mentions California as an example but doesn't suggest that the federal government should offer the exact same incentives as California did. Choice (D) is the best answer.

15. B. The author has no objection to enforcing conformity on public school students.

The author obviously doesn't think public schools should be a forum for individual expression, so Choice (A) is wrong. The author clearly doesn't object to conformity, so Choice (B) may be right. Choice (C) isn't right because the author mentions dress codes in addition to uniforms. Choice (D) is wrong. The argument doesn't contain any evidence about parents’ opinions of dress codes. Choice (E) is wrong. The author admits dress codes and uniforms limit individual expression and apparently doesn't care. Choice (B) is the best answer.

16. D. jumps to the conclusion that the defect in the glasses must be due to the optometrist's lack of skill

The conclusion is that the optometrist is incompetent; the evidence is that one lens pops out regularly. But there's no evidence that that's because of the optometrist's lack of skill. Choice (A) is wrong. Although giving the optometrist a chance to defend himself would be nice, it's not a fault of the argument that the speaker doesn't provide one. Choice (B) is wrong because other potentially unskilled optometrists have no bearing on the skills of the one in question here. Choice (C) doesn't work. The author doesn't mention any particular techniques. Choice (D) may be the answer. The author does jump to a conclusion here without making a connection between the glasses and the optometrist's skill. Choice (E) is wrong because the author doesn't suggest that sabotage played a role in the bad glasses. Choice (D) is the best answer.

17. D. A mother threatens to take away her son's toy airplane if he throws a tantrum that afternoon. He throws a tantrum that afternoon, so she takes his toy airplane.

Look for an instance of someone carrying out a promise upon the successful completion of the specific act upon which that promise was predicated. Choice (A) is wrong because the woman wasn't asked to perform anything to receive the ring. Choice (B) is wrong because the boss gives the workers something other than what he promised. Choice (C) is wrong because the seal trainer makes no promise to the seals. Choice (D) is correct. The mom promises to do something (take away a toy) if her son does a particular action (throws a tantrum); he does it, so she keeps her promise. Choice (E) is wrong because the professor didn't promise a bad recommendation for a bad performance. Choice (D) works the best.

18. E. The meadow voles that had the prairie vole gene implanted in them were released into and observed in the same habitat in which they had previously lived.

Look for information that supports the assumption that the meadow voles’ change in behavior was caused by the implanted gene. Choice (A) is wrong. The choice doesn't relate the effects of the hormone to the gene that makes meadow voles monogamous. Choice (B) explains what's up with prairie voles but not with meadow voles, and neither's genes are mentioned. Choice (C) explains why meadow voles are typically promiscuous but says nothing about whether a gene plays a part in that. Choice (D) says nothing about whether the transferred gene is the cause of the monogamous behavior. Choice (E) provides the most support for the assertion that the scientist's work with genes was the factor that turned the formerly promiscuous meadow voles into models of monogamy because it rules out a possible other important factor that may have explained the change (different surroundings). Choice (E) is the correct answer.

19. A. The art museum cannot be confident that this statue was made by Praxiteles.

Choice (A) looks like a good answer. The museum doesn't know for sure that this statue is by Praxiteles even though it has some evidence to hope it is. Choice (B) doesn't work because the passage never mentions modern forgers, only ancient Roman copiers. Choice (C) is wrong. The passage says that dating ancient sculptures accurately is difficult, which means scholars may not ever be able to date this one conclusively. Choice (D) is too specific to be a proper conclusion drawn from the passage. Choice (E) is wrong. The passage tells you that plenty of Roman copies have been mistaken for originals. Choice (A) is the best answer.

20. C. the new terminology guidelines were a valuable and necessary update to the field of psychological research

Psychologist A likes the change in practices referring to research participants; Psychologist B doesn't because it makes his life complicated. Choice (A) is wrong because they're not arguing about the preferences of participants; neither psychologist takes participants’ preferences into account. Choice (B) is wrong because both psychologists seem to agree that the governing body has the authority to make these changes. Choice (C) looks right. They do disagree about whether the changes were a good thing. Choice (D) isn't right. Psychologist A mentions children's ability to consent, but Psychologist B doesn't address that issue at all, so you can't say that they disagree about it. Choice (E) is wrong because Psychologist B doesn't address this claim, so you don't know what he thinks about it (he could actually agree that participant is a more active label and still prefer the simplicity of subject). Choice (C) is the best answer.

21. C. Women who want to have children increasingly seek to delay doing so for many varied reasons.

The conclusion is that predicting when menopause will occur will make a difference to women planning when to have children, which must mean that not knowing when menopause will occur makes it difficult to plan. Choice (A) is wrong because it doesn't explain why predicting menopause will help anyone. Choice (B) just provides general information about menopause. Choice (C) may be right — it provides a reason that women would benefit from knowing when they will experience menopause (they're delaying longer, so they need to know how long is too long to delay). Choice (D) isn't relevant because the argument is about how accurately predicting the onset of menopause affects childbearing decisions, not how likely a woman is to conceive in the years immediately prior to menopause. Choice (E) is just information about ovaries, not an explanation of how this test will help make family planning decisions. Choice (C) is the best answer.

22. C. fails to consider that funding levels are not the only factors influencing school performance

The school board member assumes that the inner-city school's poor results are due to it squandering its money, which isn't necessarily the right conclusion; other factors may be involved. Choice (A) is wrong. The member isn't suggesting that funding is tied to graduation rates, because both schools got the same funding. Choice (B) is wrong because you can't necessarily assume the school board member thinks that the 90 percent of inner-city students that don't go to college were necessarily unprepared for college. Choice (C) looks like a possible answer. The school board member assumes that having equal budgets should mean that the schools’ students should perform equally and fails to consider other possibly relevant differences between the schools, such as their facilities, their teachers, their level of parental involvement, and so on. Choice (D) isn't quite right. He's not directing criticism specifically at the administrators. Choice (E) is wrong. He's blaming the inner-city schools, not the inner-city students. Choice (C) is correct.

23. A. My European client calls once a week, always in the evening, after everyone has left the office. I'll be sure to get his messages if I turn on my telephone's answering machine once a week.

The flaw in the argument is the mistaken belief that the odds of an event occurring can tell you how often you need to do a certain act. Odds of 1 in 1,000 don't mean that every 1,000th trip will realize a certain event. It means that an accident could happen in any trip out of 1,000, and you can't predict which one. The flawed reasoning in Choice (A) is similar to this; turning on the answering machine on just one particular day won't necessarily catch a weekly phone call because the call could come on any day of the week. Choice (B) is wrong. The conclusion is mistaken but in a different way from the original argument. It's about proportionality not probability. Choice (C) isn't exactly the same as the original argument because you're not trying to guess which one of the 1,000 games will result in the jackpot; instead you're covering them all. That's actually closer to wearing the helmet for all 1,000 rides on the assumption that one of them will involve a wreck. Choice (D) is totally wrong because the second sentence is nothing like the original argument's conclusion; it doesn't state how many times people in cars should wear seat belts based on seat belt statistics. Choice (E) is flawed but not in the same way as the original argument. The flaw would be more similar to the original argument if the law student applied to only one of 20 law schools because the odds are 1 in 20 of being chosen. Choice (A) is the closest and is correct.

24. A. By the 1950s, people expected cleaner clothes and surroundings and more elaborate meals than they had earlier in the 20th century.

Choice (A) looks like an ideal answer. If standards went up, women could well spend more time on housework even with their labor-saving devices. Choice (B) is irrelevant; it says nothing about how long women spent on housecleaning. Choice (C) looks at first like it may explain the discrepancy, but the argument doesn't mention childcare, only housework. Choice (D) would work if it suggested that prewar husbands had helped their wives, but it doesn't. Choice (E) is irrelevant. Choice (A) is the best answer.

25. D. whether consumers think that geographical food names are generic

The two unions disagree about the EU's ban on using regional names for non-regional products, with the AFMU claiming that some regional names now have generic, not regional, meaning associated with them. Choice (A) is wrong because the unions aren't arguing about quality. Choice (B) is wrong because they're not discussing imports. Choice (C) is completely off; they're not debating taste buds. Choice (D) summarizes the crux of the disagreement. It looks like the right answer. The EU thinks generic names breed confusion; the AMFU thinks they don't. Choice (E) isn't right because they're not really debating intellectual property law in theory as much as addressing one specific instance of it. Choice (D) is the best answer.

Section IV: Analytical Reasoning

Questions 1–5

You don't need a game board to answer the first question in this ordering set, but you do for the remaining questions. To create an accurate game board, first make a list of the initials of the participants’ last names: A, D, H, M, S, and W. (This step is especially important in this problem because the participants all have such long names.) Write out the speaking times in a row with space under them for plugging in data; go ahead and note that the 6 o'clock time slot can't contain either D or M. Then write the other rules and their contrapositives in shorthand under your game board.

Make additional deductions based on the rules. You know that S must speak no later than 6 because she must speak before D or M. You also know that S can't speak first in the 2 slot because either D or M must speak before her. Your initial game board will therefore look something like this:

9781118678237-un18001.tif

At this point, you could stop and consider the effects of a couple of possible scenarios. If D speaks at 4 and M speaks at 7, S must speak at 5 or 6. That's mildly interesting. You also know that if H speaks at 2, then S can't speak at 4, because then W would have a conflict between speaking at 7 (the last slot) and speaking before A. The rules really are quite explicit, so get going on the questions.

Just be aware that the numbering starts at 2 p.m. Many hapless students forget that the person speaking at 2 speaks first!

1. D. Jane Hunt, Martha Wright, Frederick Douglass, Elizabeth Cady Stanton, Susan B. Anthony, Lucretia Mott

Eliminate answers that contain rule violations. The first rule specifies that Douglass and Mott don't speak at 6. Choice (B) puts Mott in the 6 spot, so it must be wrong. The second rule states that Stanton can't speak before both Douglass and Mott, so Choice (E) is out. The third rule applies when Hunt speaks first. Choices (C) and (D) put Hunt in the first spot, but Choice (C) fails to put Anthony after Wright. The last rule applies when Stanton speaks at 4. Only Choice (A) has Stanton speaking at 4, but Choice (A) doesn't place Wright in the last speaking time at 7, so the only answer that doesn't violate a rule has to be Choice (D).

2. E. Martha Wright speaks at 5.

Record the temporary condition provided by this question on your game board. If Hunt speaks at 2, Wright must speak before Anthony, which means Wright can't speak last at 7. If Wright isn't in the 7 slot, Stanton can't be in the 4 slot. Stanton can't speak at 3 or 5 because then she'd have to speak before both Douglass and Mott. So Stanton must speak at 6. With Stanton in the 6 slot, you know that either Douglass or Mott must be in the 7 slot. Because Wright has to speak before Anthony, Wright can't be in the 5 slot and Anthony can't be in the 3 slot. You could write out the possibilities for this question like this:

9781118678237-un18002.tif

Now you can see that Choices (A), (B), (C), and (D) could work, so cross them out. That leaves Choice (E), which has to be false because it doesn't allow for three time slots after Wright to fit in Anthony, Stanton, and either Mott or Douglass. Choice (E) is correct.

3. D. At least one person speaks before Jane Hunt speaks.

When you record the temporary condition that Wright speaks at 7 to your game board, what do you know? The contrapositive to the third rule reveals that Hunt can't speak at 2 when Wright doesn't speak before Anthony. Check the answers. If Hunt can't occupy the 2 slot, at least one other person must speak before Hunt. Choice (D) says just that; one person must speak before Hunt. When you consider the possible scenarios, you realize that the other answer choices are either false or merely possible:

9781118678237-un18003.tif

So Choice (D) is the only choice that must be true.

4. D. Elizabeth Cady Stanton

You can answer this question with a quick look at your original game board. Who can't occupy the first slot? Stanton; she can never speak at 2. The answer is Choice (D).

5. A. Susan B. Anthony at 3; Elizabeth Cady Stanton at 6

This problem is another rule-violation question. Plug the speakers into the time slots suggested by each answer choice and see whether they work. Choices (B) and (C) place Hunt in slot 2. You know from your work on Question 2 that when Hunt speaks at 2, Stanton can't speak at 3 and Wright can't speak at 7. Cross out Choices (B) and (C). Check the other answer choices with known restrictions. Choice (D) places Stanton in the 4 slot. When Stanton speaks at 4, Wright has to speak at 7. Choice (D) doesn't work. Consider Choice (E). If Stanton speaks at 5 and Anthony at 7, no slot is available for either Douglass or Mott to speak after Stanton, because neither of them can speak at 6. What about Choice (A)? You can schedule the speakers in the following order: Wright, Anthony, Hunt, Douglass, Stanton, Mott. Choice (A) is the only possible scenario.

Questions 6–12

First, create a game board for this grouping set. Make a list of the game pieces: the initials of the four women — M, N, O, and P — and three men — A, B, and C. Then create a game board that orders the dance events.

Consider the rules. You know that every event must have one member and that no member competes more than once. Every event with a pair of dancers must include one man and one woman. A dances alone, so the maximum number of pairs is two, and two women must dance solo. What else? Well, if O competes in the rumba, then A can't. Because N must be part of a pair, she must dance with B or C. You also know that the two pair events can't be back-to-back because B and C can't dance in consecutive events.

Record that what you know: There are two pairs and three solos; N is paired, which means you only need one more pair! Your main diagram may look something like this:

9781118678237-un18004.tif

6. E. Natasha competes in the waltz.

Record the temporary conditions imposed by this question on your game board. When Phoebe dances solo in the foxtrot and Bertrand provides a partner for Olivia, only one event is left for Christophe to compete in — the waltz. He can't do the paso doble or tango without being immediately before or immediately after Bertrand. Natasha must dance with Christophe because she needs a male partner, and he's the only one available. So Natasha must also dance the waltz. Chart what you know on the game board:

9781118678237-un18005.tif

The answer is Choice (E).

7. C. three

You're looking for the maximum number of events for Natasha, so start high and work your way down. Because Antonio dances alone, Natasha won't have five events to choose from. Eliminate Choice (E). Olivia's already in the rumba slot, so four isn't possible. Cross out Choice (D). Could it be three? Try Antonio in the foxtrot. That leaves the paso doble, tango, and waltz open. Natasha could feasibly compete in any of those, as long as she dances with Bertrand or Christophe. Choice (C) is the answer.

8. A. Antonio

If solo women compete in the foxtrot and paso doble, they must be Madeleine and Phoebe because Olivia is booked for the rumba and Natasha dances as part of a pair. The only dance left for Antonio is the tango because Olivia and either Christophe or Bertrand are competing in the rumba. The only spot left for the other member of the Christophe-Bertrand duo is the waltz with Natasha. Your game board for this question looks like this:

9781118678237-un18006.tif

The answer must be Choice (A).

9. B. Christophe, Bertrand, and Natasha

You know that both Bertrand and Christophe must dance with partners because you need two pairs with one man in each and Antonio dances solo. The only answer that contains both Christophe and Bertrand is Choice (B).

10. E. Madeleine, Olivia, and Phoebe

When you know that two women must dance solo, you know the answer can't be Choice (A) or Choice (B). Unfortunately, none of the other answers include Natasha, who would be a dead giveaway that the answer was wrong. You already know from Question 9 that both Madeleine and Phoebe can compete solo, so Choice (C) has to be wrong. To choose between Choices (D) and (E), ask yourself whether Olivia can compete solo. Sure — if, for example, Phoebe and Christophe dance the paso doble and Natasha and Bertrand compete in the waltz, Olivia is on her own for the rumba. So the correct answer is Choice (E).

11. B. foxtrot and tango

Add the temporary conditions to your game board. When Natasha dances the waltz and the females dance consecutively, the females must dance the paso doble, rumba, tango, and waltz. That puts Antonio alone in the foxtrot. You can eliminate any answer choice that doesn't include the foxtrot: Choices (C) and (E). Bertrand or Christophe must compete in the waltz with Natasha because Natasha needs a partner. The remaining Bertrand or Christophe can't dance the tango right before the waltz, so he must dance either the rumba with Olivia or the paso doble with Madeleine or Phoebe. That means a solo woman must perform the tango. Either the paso doble or the rumba will be the third solo, but you don't know which. Your game board reveals the two possible arrangements of the solo events:

9781118678237-un18007.tif

So the foxtrot and the tango are the only events that must have solo performers, and the answer is Choice (B).

12. E. Natasha competes in the foxtrot.

Add the temporary rule that the three men dance the first three dances to your game board. Neither Bertrand nor Christophe can dance the paso doble because they can't dance consecutively. So Antonio dances the paso doble. You also know that Natasha must dance the foxtrot with either Bertrand or Christophe. Your diagram for this question looks like this:

9781118678237-un18008.tif

Natasha must dance the foxtrot, so the answer is Choice (E).

Questions 13–18

Organizing this complicated ordering problem can be challenging. Consider that you have three categories of information:

·        Times the courses can be served

·        The courses themselves

·        The chefs who can prepare them

To solve the accompanying questions, first make a list of your game pieces. Use lowercase initials for courses (h, c, e, s, and d) and uppercase initials for chefs (A, B, E, J, K, M, and N). Two things remain consistent: the order of the five courses and the three serving hours of the dinner. The serving hours stay in one order, but you don't know how many courses will occur at each one (though you do know it'll be one or two, not zero or three). That means that at 7 the kitchen could serve just hors d'oeuvres or it could serve hors d'oeuvres and consommé (which are, by the way, appetizers and a kind of soup). At 8, the kitchen could serve consommé and the entrée, just the entrée, or the entrée and salade. (Salade is the fancy French word for salad, but you already figured that out.) At 9, the kitchen will serve either salade and dessert or just dessert. Which course happens when depends on the other information in the questions. Assume the kitchen has enough room for two chefs to prepare different courses at the same time without interfering with each other. You can also assume that if two courses occur in one hour, they're served one after the other, not simultaneously. That must be the case if the courses are to occur in order.

Remember to leave your preconceived notions at the door! You may know that salad almost never comes after the entrée in a 5-star restaurant, and of course dessert always comes last, but that's not important here. What's important is what the facts say.

Three scenarios are possible, so make your life easier and just draw a game board to illustrate all three:

9781118678237-un18009.tif

When you plug in your data, just write the initials of the chefs under the course they're preparing. Be sure you have enough room.

Now, remember that though the same chef can prepare more than one course, no chef can prepare two courses the same hour. So if both hors d'oeuvres and consommé are served at 7, the same chef can't prepare them both. For example, if both consommé and the entrée come out at 8, Andrea can't prepare them both, even though he's offered to. (And yes, in this case, Andrea is a guy — he's Italian.)

Make yourself another little set of notes with abbreviations of courses and chefs. Doing so makes reading the abbreviations easier than the text version. Draw this new chart off to the side of your game board for quick reference:

9781118678237-un18010.tif

13. B. three

You already know you can't have more than five chefs because there are only five courses and no chef wants to share glory with another. So Choice (E) has to be wrong. You're looking for a minimum number, so start with the smallest choice. Can you schedule a dinner with just two chefs cooking every course? To do that you'd have to find one chef who can cook three courses. Skim through your list of courses and chefs to see whether you can find any chef whose initial appears three times. Nope. No single chef can cook all three courses, so two is impossible and Choice (A) is wrong. Can you produce the whole dinner with just three chefs? Take, for example, the scenario in which dessert is the only course served at 9. You could come up with a scenario in which Berthe and Andrea cook both courses at both 7 and 8, leaving dessert to a third chef; that scenario would look like this:

9781118678237-un18011.tif

So you could do the entire dinner with just three chefs. The answer is Choice (B).

14. A. Andrea and Kimiko

Include on your game board the temporary conditions the question provides. When salade comes at 9, so does dessert, so at 8 just the entrée or both consommé and the entrée are served. The group of chefs who can prepare these courses include Andrea, Kimiko, and Nigel, which means you can eliminate Choices (B), (C), and (E). Is there any reason you can't serve two courses at 8? No, which means you can eliminate Choice (D). To verify that Choice (A) is the answer, consider the three ways the 8 o'clock course could be served:

9781118678237-un18012.tif

One of those choices, Andrea and Kimiko, happens to be the answer.

15. E. Berthe, Emilio, Jacques, and Marthe

Eliminate answers that are possible complete lists of chefs who could prepare the entire meal. Cross out Choice (A) because it could work. You could have

9781118678237-un18013.tif

Here's one possibly for Choice (B):

9781118678237-un18014.tif

You can arrange these chefs in a couple of other ways, but you need only one to eliminate Choice (B). To eliminate Choice (C), you can consider this possibility:

9781118678237-un18015.tif

Here's a possibility for Choice (D):

9781118678237-un18016.tif

That leaves Choice (E). Does leaving out Andrea make such a difference? Yes. Without Andrea and without Kimiko, you have no one willing to make consommé. Nor do you have anyone to make the entrée; Andrea and Nigel are the guys for that, and neither of them is in the list. A superabundance of chefs to make hors d'oeuvres, salade, and dessert doesn't make for a lengthy or complete banquet. That means the answer that doesn't provide a complete and accurate list is Choice (E).

16. A. Andrea and Berthe

There are three ways this question could work; the two chefs can prepare the courses at 7 and 8, 8 and 9, or 7 and 9. Take a look at the answer choices; Andrea appears three times, so he's a good one to start with. The only way Andrea can prepare two courses is if he prepares consommé at 7 and the entrée at 8. Is there another cook who could prepare hors d'oeuvres at 7 and salade at 8? Yes. Berthe could do both, leaving dessert for someone else. This is the scenario you came up with in Question 13. That means the answer is Choice (A). (Try out the other possibilities yourself; you'll see that they don't work.)

17. C. Berthe and Kimiko

The only way for this question to work is to have hors d'oeuvres and consommé at 7 and salade and dessert at 9. Look at the list of courses with their corresponding chefs. Is there anyone who can make hors d'oeuvres and either salade or dessert? Yes. Berthe can do hors d'oeuvres and salade. Is there a chef who can do consommé and dessert? Yes, Kimiko. That's one of the possible answers: Choice (C).

9781118678237-un18017.tif

None of the other pairs work; test them to see, but Choice (C) is the answer.

18. A. Andrea, Berthe, Emilio, and Nigel

This question is just like Question 15 — remember how much fun that was? This one, though, works the other way. Four of the answer choices are wrong and only one is correct; if one course is missing, then the answer is wrong. Try Choice (A). It looks like it should work. Try arranging the chefs like this:

9781118678237-un18018.tif

Test the other four to make sure. Choice (B) doesn't work because no one on its list of five chefs can make hors d'oeuvres. Choice (C) has no one to make an entrée. Choice (D) has no one to make consommé. Choice (E) doesn't have anyone who can make an entrée. So Choice (A) is the only answer that works.

Questions 19–25

The first question in most logic game sets merely requires you to apply the rules to each answer. But coming up with a good, clear game board is the key to figuring out the solutions to the remaining questions in the set. First things first — set up the game pieces using uppercase initials for the movies (E, G, I, M, and W) and lowercase letters to mark film format (v for videotape and d for disc). You may also want to add two Xs to designate the two empty days. So, you designate Emma on videotape as Ev and Wuthering Heights on disc as Wd.

Draw a table on your game board with the nights of the week as column heads. Start on Friday and finish with Thursday. Note that there are seven nights and five films, so two nights are empty. Record the rules on your game board. The easiest rule to write down is Ev in the Monday column. The second rule states that the student watches Wd before Ev, so the student can't watch Wd on Tuesday, Wednesday, or Thursday nights. Because the student watches G before both M and I, he can't watch G on Wednesday or Thursday, nor can he watch M or I on Friday.

Consider the ramifications of the final rule. If the student can't watch any two videotapes or two DVDs next to each other, he can't watch any film on Sunday. Sunday must be an empty day. You also know that he must watch W on Saturday because that's the only way he can watch it before E without violating the final rule. You can draw a further conclusion: If the student watches a movie on Friday, it must be a videotape.

See whether you can come up with more information about the other films and nights. If the student views a film on Tuesday, it must be on disc. You also know that Great Expectations must come before both Ivanhoe and Middlemarch, but you don't know the order of Ivanhoe and Middlemarch. You can make that work in two ways — either Great Expectations is the video on Friday or it's a disc on Tuesday. If Great Expectations is a video scheduled for Friday, then Ivanhoe and Middlemarch are discs on Tuesday and Thursday. If instead Great Expectations is a disc on Tuesday, then either Ivanhoe or Middlemarch is a videotape, and the other of the pair is a disc. In that case, the student would distribute Ivanhoe and Middlemarch in the proper order between Tuesday and Thursday. Note that the only possible film for Friday is Great Expectations on video (Gv), so if the student watches a film on Friday, that's the one. That means the two possible schedules look like this:

9781118678237-un18019.tif

When you narrow down the possible schedules to these two, answering the questions is a piece of cake.

19. C. Friday: Great Expectations; Saturday: Wuthering Heights; Monday: Emma; Tuesday: Middlemarch; Thursday: Ivanhoe

You can usually answer the first question in the set by eliminating answer choices that violate the rules. The first condition states that the student watches Wuthering Heights before Emma. No choices violate this rule. The next condition specifies that the student watches Middlemarch and Ivanhoe after Great Expectations. Choice (E) violates the rule by scheduling Middlemarch before Great Expectations. Eliminate Choice (A) because it schedules Emma on Sunday. Choice (D) schedules Wuthering Heights on Friday and no film on Saturday, which violates the rule that the student watches a film on disc for Saturday. To determine whether Choice (B) or Choice (C) is correct, apply the fifth condition that a video and disc can't be watched on consecutive nights to both. Choice (B) schedules a disc (Wuthering Heights) on Saturday, which means that the film the student saw the night before (Great Expectations) must have been a video. Emma is also on video, so that takes care of the two films on videotape and means that the student views discs on two consecutive nights, Tuesday and Wednesday. That can't be. Choice (C) inserts an off day between the two films that must be on disc, so it's the only answer that doesn't violate a rule.

20. A. Great Expectations is on disc.

The only way the student can view Great Expectations before Emma is to watch it on Friday (the first schedule listed on your game board) because Wuthering Heights occupies Saturday and Sunday is off. The film on Friday must be on videotape; otherwise, it would violate the rule about putting films of the same format on days immediately before or after each other. So Great Expectations can't be on disc, and the answer is Choice (A). As for Ivanhoe and Middlemarch, either one of them could be on disc. You know there's no film on Sunday, and they could take place on Tuesday and Thursday, so the other four answers either must be or could be true.

Choice (A) is correct.

21. C. The student watches Great Expectations exactly one day before watching Middlemarch.

If Middlemarch is on videotape, the student must be watching the films according to the second schedule on your game board. So Great Expectations and Ivanhoe both must be on disc. In that case, Great Expectations must be on Tuesday, Middlemarch on Wednesday, and Ivanhoe on Thursday, with no film on Friday.

Of the answer choices, only Choice (C) is possible. Choice (A) is wrong because Ivanhoe is on disc. Choice (B) is wrong because the student must watch Emma two days before watching Middlemarch. Choice (D) is wrong because the student must watch Ivanhoe on Thursday. Choice (E) is wrong because the student must watch Middlemarch on Wednesday. But he must watch Great Expectations exactly one day before watching Middlemarch,so Choice (C) is correct.

22. E. The student doesn't watch a film on either Tuesday or Thursday.

Test the answers, checking them on your game board. Eliminate those that either must be or could be true. Try Choice (A). According to the first schedule, the student could watch G on Friday and E on Monday, so that could be true. Now take a look at Choice (B). He must watch a film on Monday, and Wednesday can go either way, so it could be true that he watches films on Monday and Wednesday. Try Choice (C). He could take Friday and Sunday off, so that could be true. Consider Choice (D). He definitely skips Sunday and could take Wednesday off if he watches Great Expectations on Friday, so it could be true that he doesn't watch on Sunday or Wednesday. That leaves Choice (E), and it's the only false statement. Every possible schedule on the game board requires the student to watch something on either Tuesday or Thursday. The answer is Choice (E).

23. C. The student watches Great Expectations before he watches Emma.

If the student watches Great Expectations after Wuthering Heights, he's watching films according to the second schedule on the game board. He must watch Great Expectations on disc on Tuesday, and Middlemarch and Ivanhoe must fall on Wednesday and Thursday, with the Thursday film on disc and the Wednesday film on videotape. Remember, either Ivanhoe and Middlemarch can be viewed on Wednesday or Thursday. The student can't watch anything on Friday, so Choice (A) must be true. Choice (B) could be true if Ivanhoe is on videotape, which would put it on Wednesday. Choice (C) must be false because Emma must be on Monday and Great Expectations must be on Tuesday; that looks like the answer. Choice (D) must be true because Great Expectations must be on disc. Choice (E) could be true because Ivanhoe could be on disc. Choice (C) is correct.

24. A. The student watches Great Expectations on Friday.

With five movies and seven nights to watch them, the student can take only two nights off. One of those is Sunday; the other could be Friday or Wednesday. If he takes Wednesday off, then the first schedule on your game board applies, and he has to watch a film on Friday. The only film he can watch on Friday is the videotape of Great Expectations. That means Choice (A) must be true, and Choice (B) is false. Choices (C), (D), and (E) are all possible but not necessarily true.

Choice (A) is the answer.

25. B. two

The schedule for Saturday, Sunday, and Monday is the same for every schedule on your game board. The day the student sees Ivanhoe depends on when he sees Great Expectations. If he watches Great Expectations on Friday, then Great Expectations is a videotape and Ivanhoe and Middlemarch are both discs. To watch Ivanhoe before Middlemarch and avoid viewing two discs on consecutive days, the student must watch Ivanhoe on Tuesday and Middlemarch on Thursday. If, on the other hand, he watches Great Expectations on Tuesday, Great Expectations must be on disc, and his Thursday movie must also be on disc. His Wednesday film — in this case, Ivanhoe — must be on videotape.

So the answer is Choice (B).

Example Essay

The law student’s dilemma is whether he should sign on with a small boutique law firm or a larger one given his desires to both explore many different areas of legal practice and take on early responsibilities in hopes of advancing quickly within a company. While working at the smaller boutique firm may allow the student to advance more quickly, the student would be limited there in terms of what areas of law to explore. The larger firm would enable the student to explore all types of law, but it has limitations on how quickly he may move up the corporate ladder. While neither of the two options will allow the student to fully satisfy both goals, the student’s better option is the latter. The smaller firm cannot guarantee the student early advancement, and working in the large firm would allow the student to try out a number of different legal career paths in his quest to find the right fit for him.

Should the student select the larger firm, he is more likely to uncover the area of law he would like to pursue and fulfill his goal of experiencing a variety of legal topics. Exploring a new area every six months will provide the student with ample time to realize which specialization he prefers. This experience would not be available at the small firm, where he would focus largely on document-heavy areas of law. He would gain little trial or litigation experience. The larger firm provides the student with a better environment within which to explore a variety of legal specializations.

This diversity of experience requires the student to sacrifice the potential for early partnership, however. The earliest he could make partner at the larger firm would be after seven years of employment. The smaller firm makes advancement possible around four years earlier. While the smaller firm may provide the student a better opportunity to take on more responsibility, quick advancement up the ladder is not guaranteed. The hiring partner at the boutique firm specifically said that he could be a partner in three to four years if “he learns quickly and gets along with his coworkers.” Regardless of how fast a learner he may consider himself, the student cannot be sure that he will get along with his coworkers. This factor is to some degree beyond the student’s control; it is somewhat dependent on the personalities of his coworkers. Furthermore, the prompt states that partnership at the larger firm cannot be achieved before seven years, but that is not to say there would not be other methods of excelling within the firm in the interim. Working in teams with a number of attorneys will give the student mentorship and contacts to promote a smooth advancement after he settles on and masters his chosen field of practice.

Therefore, because neither option adequately addresses both of the student’s desires, the second option, signing on with the larger firm with the wider range of practices, offers the student a better opportunity for lifelong career satisfaction and success. The larger firm provides the student with exposure to and practice in many areas of law, connections with many different attorneys, and mentorship, factors that outweigh the available but not guaranteed earlier promotion at the more limited smaller firm.

Answer Key for Practice Exam 2

Section I: Logical Reasoning

1. B

2. E

3. C

4. D

5. D

6. A

7. B

8. B

9. E

10. A

11. E

12. C

13. C

14. E

15. E

16. D

17. C

18. B

19. A

20. D

21. B

22. E

23. A

24. E

Section II: Reading Comprehension

1. D

2. A

3. C

4. A

5. D

6. E

7. C

8. B

9. E

10. D

11. E

12. E

13. A

14. D

15. B

16. A

17. E

18. D

19. E

20. C

21. C

22. B

23. A

24. E

25. B

26. D

Section III: Logical Reasoning

1. A

2. E

3. B

4. A

5. D

6. C

7. E

8. D

9. D

10. A

11. E

12. C

13. B

14. D

15. B

16. D

17. D

18. E

19. A

20. C

21. C

22. C

23. A

24. A

25. D

Section IV: Analytical Reasoning

1. D

2. E

3. D

4. D

5. A

6. E

7. C

8. A

9. B

10. E

11. B

12. E

13. B

14. A

15. E

16. A

17. C

18. A

19. C

20. A

21. C

22. E

23. C

24. A

25. B

tip.eps Refer to Chapter 16 for tips on computing your score.